2010 AMC 12B Problems/Problem 12

Revision as of 18:51, 9 July 2010 by Lg5293 (talk | contribs) (Created page with '== Problem 12 == For what value of <math>x</math> does <cmath>\log_{\sqrt{2}}\sqrt{x}+\log_{2}{x}+\log_{4}{x^2}+\log_{8}{x^3}+\log_{16}{x^4}=40?</cmath> <math>\textbf{(A)}\ 8 \…')
(diff) ← Older revision | Latest revision (diff) | Newer revision → (diff)

Problem 12

For what value of $x$ does

\[\log_{\sqrt{2}}\sqrt{x}+\log_{2}{x}+\log_{4}{x^2}+\log_{8}{x^3}+\log_{16}{x^4}=40?\]

$\textbf{(A)}\ 8 \qquad \textbf{(B)}\ 16 \qquad \textbf{(C)}\ 32 \qquad \textbf{(D)}\ 256 \qquad \textbf{(E)}\ 1024$

Solution

Using logarithm rules: \begin {align*} \log_{\sqrt{2}}\sqrt{x} + \log_2x + \log_4(x^2) + \log_8(x^3) + \log_{16}(x^4) & = 40 \\ \frac{1}{2} \frac{\log_2x}{\log_2\sqrt{2}} + \log_2x + \frac{2\log_2x}{\log_24} + \frac{3\log_2x}{\log_28} + \frac{4\log_2x}{\log_216} & = 40 \\ \log_2x + \log_2x + \log_2x + \log_2x + \log_2x & = 40 \\ 5\log_2x & = 40 \\ \log_2x & = 8 \\ x & = 256 (D) \end{align*}

See also

2010 AMC 12B (ProblemsAnswer KeyResources)
Preceded by
Problem 12
Followed by
Problem 14
1 2 3 4 5 6 7 8 9 10 11 12 13 14 15 16 17 18 19 20 21 22 23 24 25
All AMC 12 Problems and Solutions